Download Comprehensive Nursing Exam Preparation: Mastering Clinical Assessments and more Exams Nursing in PDF only on Docsity! NR509 (3 versions) Midterm exam + NR509 Bates Test Bank - Midterm NR509 Midterm version 1 A teenage girl has arrived complaining of pain in her left wrist. She was playing basketball when she fell and landed on her left hand. The FNP examined her hand and will expect a fracture if the girl complains: - correct answer Of sharp pain that increases with movement A patient has been diagnosed with osteoporosis and asked the FNP "what is osteoporosis?" The FNP explains to the patient that osteoporosis is defined as: - correct answer Loss of bone density Patient states, "I can hear a crunching or grating sound when I kneel". She also states "that it is very difficult to get out of bed in the morning because of stiffness and pain in my joints". The FNP should assess for signs of what problem? - correct answer Crepitation When taking the history on a patient with a seizure disorder the FNP assesses whether the patient has an aura. Which of these would be the best question for obtaining this information? - correct answer Do you have any warning sign before you receive your starts? The FNP is teaching a class on osteoporosis prevention to a group of post menopausal women. A participant shows that she needs more instruction when she states I will: - correct answer " Start swimming to increase my weight bearing exercise" The FNP is performing a neurological assessment on a 41-year-old woman with a history of diabetes. When testing her ability to feel the vibrations of a tuning fork the FNP notices that the patient is unable to feel vibrations on the great toe or ankle bilaterally, but she is able to feel vibrations on both patella. Given this information what would the FNP suspect? - correct answer Peripheral neuropathy And 80-year-old woman is visiting the clinic for a check up. She states "I can't walk as much as I used to". The FNP is observing from motor dysfunction in her hip and should have her: - correct answer Abduct her hip while she is lying on her back The FNP is testing the function of cranial nerve XI. Which of these best describes the response NR509 (3 versions) Midterm exam + NR509 Bates Test Bank - Midterm the FNP should expect if the nerve is intact?The patient - correct answer Moves the head and shoulders against resistance with equal strength A 50-year-old woman is in a clinic for weakness in her left arm and leg that she noticed for the past week. The FNP should perform which type of neurologic examination? - correct answer Complete neurologic examination During the neurological assessment of a healthy 35-year-old patient the FNP asks him to relax his muscles completely. The FNP then moves each extremity through full range of motion. Which of these results would the FNP expect to find? - correct answer Mild, even resistance to movement A woman who is 8 months pregnant comments that she has noticed a change in posture and is having lower back pain. The FNP tells her that during pregnancy women have a posture shift to compensate for the enlarging fetus. This shift and posture is known as: - correct answer Lordosis A patient is able to flex his right arm forward without difficulty or pain but is unable to abduct his arm because of pain and muscle spasm's. The FNP should suspect: - correct answer Rotator cuff lesions A professional tennis player comes into the clinic complaining of a sore elbow. The FNP will assess for tenderness at the: - correct answer Medial and lateral epicondyle The FNP suspect that a patient has carpal tunnel syndrome and wants to perform the Phalens test. To perform this task the FNP should instruct the patient to: - correct answer Hold both hands back to back while flexing the wrists 90° for 60 seconds During the history a patient tells the FNP that "it feels like the room is spinning around me". The FNP would document this as: - correct answer Vertigo The assessment of a 60-year-old patient has taken longer than anticipated. In testing his pain perception the FNP decides to complete the test as quickly as possible. When the FNP applies the sharp point of a pin on his arm several times he is only able to identify these as one very NR509 (3 versions) Midterm exam + NR509 Bates Test Bank - Midterm by____ - correct answer XI; asking the patient you should have her shoulders against resistance When examining a patient's cranial nerve function, the FNP remembers that the muscles in the neck that are innervated by CN XI are the: - correct answer Sternomastoid and trapezius The patient's laboratory data reveal an elevated thyroxine level. The FNP would proceed with an examination of the _____ gland - correct answer Thyroid A patient says that she has recently noticed a lump in the front of her neck below her "Adams apple" that seems to be getting bigger. During the assessment, the finding that leaves the FNP to suspect that this may not be a cancerous thyroid nodule is that the lump: - correct answer Is mobile and not hard The FNP notices that the patient's submental lymph nodes are enlarged. In an effort to identify the cause of the node enlargement, the FNP would assess the patient's: - correct answer Area proximal to the enlarged node The FNP is aware that the four areas in the body were lymph nodes accessible are the: - correct answer Head and neck, arms, inguinal area, and axillae A 52-year-old patient describes the presence of occasional floaters or spots moving in front of his eyes. The FNP should know that floaters are usually not significant and are caused by: - correct answer Condensed vitreous fibers The FNP is preparing to assess the visual acuity of a 16-year-old patient. How should the FNP proceed? - correct answer Use the Snellen chart position 20 feet away from the patient A patient's vision is recorded as 20/30 when the Snellen eyechart is used. The FNP interprets these results to indicate that: - correct answer The patient can read at 20 feet would a person with normal vision can read it 30 feet A patient is unable to read even the largest letters on the Snellen chart. The FNP should take which action next - correct answer Shorten the distance between the patient and the chart until it is seen and record that distance NR509 (3 versions) Midterm exam + NR509 Bates Test Bank - Midterm A patient's vision is reported as 20/80 in each eye. The FNP interprets this finding to mean that - correct answer The patient has poor vision When performing the corneal light reflex assessment, the FNP notes that the light is reflected at 2 o'clock in each eye. The FNP should - correct answer Consider this a normal finding The FNP is performing the diagnostic positions test. Normal findings would be which of these results? - correct answer Parallel movement of both eyes During an assessment of the sclera of an African-American patient, the FNP would consider which of these an expected finding? - correct answer The presence of small brown macules on the sclera A 60-year-old man is at the clinic for an examination. The FNP suspects that he has ptosis of one eye. How should the FNP check for this? - correct answer Observe the distance between the palpebral fissures The FNP is doing an assessment on a 21-year-old patient and notices that his nasal mucosa appears pale gray and swollen. What would be the most appropriate question to ask the patient? - correct answer Are you aware of having any allergies The FNP is palpating the sinus areas. If the findings are normal, then the patient should report which sensation? - correct answer Firm pressure During an oral assessment of a 30-year-old African-American patient, the FNP notices bluish lips and a dark line along the gingival margin. What would the FNP do in response to these findings - correct answer Proceed with assessment knowing that this is a normal finding During an assessment of a 20-year-old patient with a three day history of nausea and vomiting the FNP notices dry mucous and deep vertical fissures on the tongue. These findings are reflective of: - correct answer Dehydration The FNP is reviewing the technique of palpating for tactile fremitus with a new graduate. Which statement by the graduate FNP reflects a correct understanding of tactile fremitus? Tactile fremitus: - correct answer It's caused by sounds generated from the larynx The FNP student is reviewing physical assessment findings of the HEENT system associated NR509 (3 versions) Midterm exam + NR509 Bates Test Bank - Midterm with pregnancy. Which statement by the graduate FNP reflects a correct understanding of expected HEENT changes associated with pregnancy? During pregnancy: - correct answer Nasal stuffiness may occur as a result of decreased vascularity in upper respiratory tract A patient tells the FNP that he is very nervous, that he is nauseated, and that he "feels hot". This type of data would be: - correct answer Subjective The patient's record, laboratory studies, objective data, and subjective data combine to form the: - correct answer Data base The FNP is reviewing information about evidence-based practice. Which statement best reflects evidence-based practice? - correct answer EBP emphasizes the use of best evidence with the clinicians experience A 59-year-old patient tells the FNP that he has ulcerative colitis. He has been having black stools stools for the last 24 hours. How would the FNP best document his reason for seeking care? - correct answer J. M. Is a 59-year-old male here for having black stools for the past 24 hours A 29-year-old woman tells the FNP that she has excruciating pain in her back. Which would be an appropriate response by the FNP to the woman's statement? - correct answer How would you say the pain affects your ability to do your daily activities? In recording the childhood illnesses of a patient who denies having had any, which note by the FNP would be most accurate? - correct answer Patient denies measles mumps, rubella, chickenpox, pertussis and strep throat If a female patient tells the FNP that she has had six pregnancies, with four live births at term and two spontaneous abortions. Her four children are still living. How would the FNP record this information? - correct answer Gravida 6, Para 4, (S) AB 2, living 4 Which of these statements represents subjective data the FNP obtained from the patient regarding the patients skin: - correct answer Denies color change The FNP is obtaining a history for a 30-year-old male patient and is concerned about health promotion activities. Which of these questions would be appropriate to use to assess health promotion activities for this patient? - correct answer Do you perform testicular self- examination's? NR509 (3 versions) Midterm exam + NR509 Bates Test Bank - Midterm c. Following the patient's lead to understand their thoughts, ideas, concerns, and requests d. Deferring respect, empathy, - correct answer c. Following the patient's lead to understand their thoughts, ideas, concerns, and requests A 17-year-old male presents to a sexually transmitted disease clinic at the behest of his brother, who convinced the patient to attend the clinic after he disclosed that he prefers homosexual partners but is afraid that his last partner may have given him an infection. The patient expresses to the intake nurse that he is unashamed of his sexual orientation and will not stay through the visit if he feels that he is dismissed or discriminated against because of it. The nurse practitioner receives this communication prior to entering the examination room and decides to employ active listening to best connect with the patient at this critical juncture in his care with the clinic. Which of the following is an example of an active listening technique? a. Ignoring visual cues to focus on the patient's exact words b. Setting aside the patient's emotional state to focus on his medical needs c. Paring down the patient's conce - correct answer d. Using nonverbal communication to encourage the patient to expand their narrative A 42-year-old female mathematician presents for follow-up care regarding a new diagnosis of systemic lupus erythematosus 6 months ago after a lengthy diagnostic process during which she was debilitated with fatigue and joint pain. Since her diagnosis, she has been minimally compliant with medications and has switched her rheumatology provider twice. She continues to feel ill, and, in explanation for her lack of adherence to the prescribed treatment, she simply says, "I don't like it." At this initial visit with her third rheumatology provider, the clinician elects to explore the issues behind her noncompliance before engaging in diagnostics and treatment using the FIFE model. Which of the following best defines the elements of the FIFE model? a. Focus, intensity, function, and evaluation b. Facts, intensity, focus, and evidence c. Feelings, ideas, function, and expectations d. Feelings, impression, fantasy, and em - correct answer c. Feelings, ideas, function, and expectations A 39-year-old nurse who is a well-established patient complains of irregular menstrual periods and pelvic pain. She says that she is having trouble sleeping and asks whether she could be NR509 (3 versions) Midterm exam + NR509 Bates Test Bank - Midterm given a "sleeping pill." The patient also says she is thinking of leaving her job. What is the best "next step" in caring for this patient? a. Perform a pelvic examination. b. Obtain a urine sample for testing. c. Obtain a more complete description of problems. d. Obtain blood for testing. e. Ask about recent travel destinations. - correct answer c. Obtain a more complete description of problems. A 29-year-old female professional athlete presents to a new primary care provider with chronic menstrual complaints. She remarks to the nursing staff that, in the past, she has experienced a dismissal of her complaints because of her high level of physical fitness and conditioning. She is seeking a care provider who will explore the issue in more detail and work with her particular concerns. Which of the following is the description of the patient-centered care this individual seeks? a. Structured and clinician-centered with open-ended questions b. Validating and empathetic with open-ended questions c. Dismissive and concrete with open-ended questions d. Affirming and reassuring with close-ended questions e. Factual and structured with active listening - correct answer b. Validating and empathetic with open-ended questions A 36-year-old female air traffic controller presents to her primary care provider for a routine visit 3 months after losing her spouse to a lengthy battle with a neurodegenerative disease. The patient denies any psychiatric symptoms on review of systems and, in fact, states that she has slept better in the last month than she had in the previous years. She endorses a healthy support system, including the extended family of her deceased spouse, with whom she is still close. She becomes wistful and briefly tearful when speaking of the plans that they had when they first married that were never fulfilled; she then changes the subject rapidly to whether her Pap smear is due. Which of the following is an example of an empathetic response to this patient? NR509 (3 versions) Midterm exam + NR509 Bates Test Bank - Midterm a. Assuming that the event caused her to become depressed and expressing the same feeling on behalf of the patient b. Recognizing the patient's emotions by asking or con - correct answer b. Recognizing the patient's emotions by asking or confirming how she feels about the event A 63-year-old male presents to establish care at a new primary care clinic to discuss issues with pain and fatigue. The clinician conducting the visit begins with general historical questions but quickly becomes suspicious that the patient is suffering from decompensated heart failure. When the patient mentions that he has had vague chest pain since last night, the clinician feels that the focus must be redirected to this potentially emergent condition. Which of the following interview techniques is the most appropriate to effectively manage this visit? a. Providing serial reassurances such as, "Don't worry, you're going to be fine." b. Asking a series of negative questions such as, "You don't have any swelling in your feet, do you?" c. Nonverbally cuing the patient to focus on his narrative regarding a motor vehicle accident d. (MVA) that led to back pain e. Asking leading questions that focus on the presumed diag - correct answer f. Moving from open-ended to focused questions A 59-year-old patient presents to his primary care provider with a history of several episodes of sharp epigastric pain. His father died of pancreatic cancer at age 52 years, and the patient recalls to the clinician that, "His pain was just like mine is now ..." The patient then pauses several seconds. The clinician replies, "Just like?" after which the patient restarts his narrative. Which of the following is an example of the interviewing techniques employed by the clinician? a. Clarifying b. Echoing c. Encouraging with continuers d. Eliciting a graded response e. Asking a leading question - correct answer b. Echoing A 14-year-old male presents to a new primary care provider after his family relocates to a state. The patient underwent treatment for sarcoma when he was age 11 years, including an above- the-knee amputation. He has learned to successfully navigate with a prosthetic leg and even engage in competitive athletics at school. He does not like to speak of his experience with NR509 (3 versions) Midterm exam + NR509 Bates Test Bank - Midterm A 21-year-old college student experiences tachycardia following a night of heavy drinking. She is advised to undergo a stress electrocardiogram (ECG). As she exercises, the recently calibrated pulse oximeter records a heart rate ranging from 25 beats per minute (bpm) at rest to 50 bpm while jogging. The test is stopped and re-started twice, and each time the pulse oximeter yields a resting heart rate of 25 and a jogging heart rate of 50. Which aspect of this instrument does the ECG technician question? a. Prevalence b. Validity c. Sensitivity d. Specificity e. Predictive value - correct answer b. Validity A 51-year-old moderately overweight college professor visits the clinic with a complaint of chest pain after tennis matches. He jokes that his tennis partner "is in a lot better shape than I am" but says that he is trying to keep up. Later in the day, a 28-year-old female student at the same college reports that "my chest often feels hot and tight." She also feels stressed on the evening before mid-term exams. The clinician recommends an immediate evaluation for coronary artery disease (CAD) for the professor, but not for the student. Why? a. Positive predictive value of an observation is higher in a group with a higher prevalence of disease. b. Positive predictive value of an observation is lower in a group with a higher prevalence of disease. c. Negative predictive value of an observation is lower in a group with a higher prevalence of disease. d. Negative predictive value of an observation is higher in a group - correct answer a. Positive predictive value of an observation is higher in a group with a higher prevalence of disease. A 23-year-old farm worker submits urine for testing. Three test strips from Container A show abnormally low pH levels in the urine. However, three test strips from Container B, purchased more recently, consistently indicate that the pH of this patient's urine is normal. Which of the following is a true statement about the test strips? NR509 (3 versions) Midterm exam + NR509 Bates Test Bank - Midterm a. The test strips demonstrate high interobserver reliability. b. The test strips demonstrate low intraobserver reliability. c. The test strips demonstrate high intraobserver reliability. d. The test strips demonstrate high sensitivity. e. The test strips demonstrate high specificity. - correct answer c. The test strips demonstrate high intraobserver reliability. A 45-year-old forklift driver presents to the clinic at 4 o'clock in the afternoon complaining of intense substernal chest pain and nausea. He appears pale and sweaty. At work that day, he filled in for an absent co-worker and was asked to perform heavy lifting not normally a part of his job. The physician assistant (PA) questions the patient in detail about his nausea, eating habits, and digestive history. Which of the following steps of clinical reasoning has the PA failed to follow? a. Elicit information about the patient's gastrointestinal (GI) system b. Localize findings anatomically c. Match findings against conditions that could cause them d. Give special consideration to potential life-threatening problems e. Elicit information about the patient's family history of digestive disorders - correct answer d. Give special consideration to potential life-threatening problems The positive predictive value of a test is calculated as the number of true positives identified by the test divided by the total positives found by the test. If a novel test for strep throat yields 150 true-positive results and 150 false-positive results, what is the positive predictive value of this test? a. 10% b. 25% c. 50% d. 75% e. 100% - correct answer c. 50% NR509 (3 versions) Midterm exam + NR509 Bates Test Bank - Midterm OLDCARTS - correct answer Onset Location Duration Character Aggravating or alleviating factors Radiation Timing Setting The negative predictive value of a test is calculated as the number of true negatives identified by the test divided by the total negatives found by the test. If a novel test for strep throat yields 85 true-negative results and 15 false-negative results, what is the negative predictive value of this test? a. 10% b. 15% c. 75% d. 85% e. 99% - correct answer d. 85% A 58-year-old carpenter presents for his annual physical examination. The physician assistant notes a systolic murmur on auscultation of the aorta. However, she does not immediately conclude that this patient has aortic stenosis. Which of the following is the reason that she seeks additional information? a. Systolic murmurs have high sensitivity and high specificity for aortic stenosis. b. Systolic murmurs have low sensitivity and low specificity for aortic stenosis. c. Systolic murmurs have low sensitivity but high specificity for aortic stenosis. d. Systolic murmurs have high sensitivity but low specificity for aortic stenosis. e. Systolic murmurs are unrelated to aortic stenosis. - correct answer d. Systolic murmurs have high sensitivity but low specificity for aortic stenosis. NR509 (3 versions) Midterm exam + NR509 Bates Test Bank - Midterm A 55-year-old air traffic control agent reports his home blood pressure log to clinic after he was diagnosed with hypertension at a prior visit. He notes that he consistently measures within the normal range at home, but seems to fall outside the normal range every time he comes to the clinic. Which of the following blood pressure measurements is considered to be most accurate (i.e., reflecting the patient's "true" blood pressure)? a. Blood pressure recorded in three positions in the health practitioner's office b. Three separate blood pressure measurements recorded by a medical technician within 90 minutes of awakening in the morning in an office setting using an automated device c. Blood pressure recorded in three positions in the health practitioner's office after resting for a 10-minute period in a supine position d. Regular ambulatory monitoring recorded outside of the office setting e. A total of six blood pre - correct answer d. Regular ambulatory monitoring recorded outside of the office setting A 68-year-old retired college professor presents for routine physical examination. After the patient has been reading a novel in the waiting room for ~20 minutes, the technician records his blood pressure in both arms using an automated device. The technician notes a 20-mm Hg difference in systolic blood pressure between the right and left arms; he repeats the readings 10 minutes later and records the same asymmetrical systolic blood pressure. Which of the following is true regarding this physical finding? a. An arm-to-arm difference of up to 20 mm Hg in systolic blood pressure is considered the upper limits of normal. b. The difference is likely secondary to white coat hypertension and should be followed up with three subsequent monthly readings to confirm. c. The patient should undergo ambulatory blood pressure monitoring in both arms for 24 hours to confirm conflicting measurements in the office. d. This finding - correct answer d. This finding is clearly abnormal and requires immediate evaluation for possible cardiovascular emergency. Position of listening to heart sounds - correct answer https://medizzy.com/feed/31152359 Aortic regurgitation -What position the patient should be in to auscultate NR509 (3 versions) Midterm exam + NR509 Bates Test Bank - Midterm -What sound will you hear - correct answer -Leaning forward -Soft diastolic decrescendo murmur Olfactory nerve: - correct answer 1 Sense of smell. Optic nerve - correct answer 2 : Ability to see. Oculomotor nerve: - correct answer 3 Ability to move and blink your eyes. Trochlear nerve: - correct answer 4 Ability to move your eyes up and down or back and forth Trigeminal nerve: - correct answer 5 Sensations in your face and cheeks, taste and jaw movements. Abducens nerve: - correct answer 6. Ability to move your eyes. Facial nerve: - correct answer 7 Facial expressions and sense of taste. Auditory/vestibular nerve: - correct answer 8 Sense of hearing and balance. ?? - correct answer 9 Vagus nerve: - correct answer 10 Digestion and heart rate. Accessory nerve (or spinal accessory nerve) - correct answer 11 Shoulder and neck muscle movement. Hypoglossal nerve: - correct answer 12 Ability to move your tongue. Optic nerve Optic neuritis - correct answer Central and peripheral vision (2) Optic neuritis: Sudden visual loss that is unilateral and can be painful, associated with multiple sclerosis FIFE - correct answer Helps to explore patient's perspective about health/illness Feelings, ideas, function, and expectations Silent patient - correct answer Silence can be therapeutic, reflection Differential diagnosis - correct answer List of possible diagnoses supported by problem list Pertinent negative - correct answer Absence of a sign or symptom that helps identify a patient's condition Pertinent positives - correct answer Specific symptoms that raise the physician's suspicion for a particular disease NR509 (3 versions) Midterm exam + NR509 Bates Test Bank - Midterm Chronic pain - correct answer Pain not associated with cancer or other medical conditions that persists for more than 3 to 6 months, pain lasting more than 1 month beyond the course of an acute illness or injury, or pain recurring at intervals of months or years. Chronic pain is the leading cause of disability and impaired performance at work. Inquire about the effects of pain on the patient's daily activities, mood, sleep, work, and sexual activity. Depressive, somatoform, and anxiety disorders affect the patient's coping strategies and have to be identified in order to effectively treat acute pain, in particular, chronic pain. Visual acuity 20/100 - correct answer Means that at 20 feet the patient can read a print that a person with normal vision could read at 100 ft, the larger the second number the worse the vision. First # indicates the distance from the chart. Funduscopic exam - correct answer The use of an ophthalmoscope to look through the pupil and examine the interior surface of the posterior eye Signs of increased intracranial pressure - correct answer •Papilledema of the optic disc leads to elevated ICP causes intraaxonal edema along the optic nerve leading to engorgement and swelling on the optic disc -Pink, hyperemic, loss of venous pulsations, disc more visible, disc swollen with blurred margins, physiologic cup not visible •Headache, blurred vision, feeling less alert than usual, vomiting, changes in behavior, weakness or problems with moving or talking, lack of energy or sleepiness Edema scale - correct answer • 1+ Mild pitting, slight indentation, no perceptible swelling of the leg • 2+ Moderate pitting, indentation, subsides rapidly • 3+ Deep pitting, indentation remains for a short time, leg looks swollen • 4+ Very deep pitting, indentation lasts a long time, leg is very swollen Macular degeneration - correct answer Macular degeneration is an important cause of poor central vision in older adults. NR509 (3 versions) Midterm exam + NR509 Bates Test Bank - Midterm b. Chronic pain is defined as pain not due to cancer or a recognized medical condition that persists for >3-6 months. c. Chronic pain is defined as focused pain lasting >8 months following acute injury or illness.d. In primary care practices, non-cancer-related chronic pain is seen in <10% of patients. e. Pain th - correct answer b. Chronic pain is defined as pain not due to cancer or a recognized medical condition that persists for >3-6 months. Disparities in pain treatment have been well described in numerous studies comparing Caucasian patients to those of African American and Hispanic origin. Which of the following statements is true concerning this issue? a. Racial and ethnic biases are only relevant in geographic areas that have a history of racial and ethnic discrimination. b. Racial and ethnic biases never involve two persons of the same race or ethnic group. c. Language barriers do not contribute to the problem of racial and ethnic biases. d. Biases of the treating clinician are associated with overtreatment of pain in minority patients and non-English speakers. e. Biases of the treating clinician are associated with under-treatment of pain in minority patients and non-English speakers. - correct answer e. Biases of the treating clinician are associated with under- treatment of pain in minority patients and non-English speakers. Which of the following statements is true concerning mental health disorders in primary care? a. The prevalence for mental disorders is estimated to be ~10%, of which only 25% are not diagnosed. b. Anxiety disorders are the most prevalent of all diagnoses in this setting. c. Somatic symptom disorder (DSM-5) is distinctly uncommon in this setting and constitutes less than 5% of these disorders. d. Mood disorders make up ~25% of all diagnoses. NR509 (3 versions) Midterm exam + NR509 Bates Test Bank - Midterm e. Alcohol and substance abuse are not considered mental health disorders.d. Mood disorders make up ~25% of all diagnoses. - correct answer d. Mood disorders make up ~25% of all diagnoses. Which of the following complaints/findings is considered to be a patient identifier for mental health screening? a. High use of health services due to chronic unstable medical diagnoses b. Symptoms lasting for >2 weeks c. Acute pain syndromes of 10 days' duration that require opiates for relief d. Substance abuse e. A patient with type I diabetes and neuropathic pain - correct answer d. Substance abuse The focused cardiovascular assessment includes - correct answer Inspection, palpation, and auscultation of the chest, neck, and extremities. Percussion is used to determine the borders of the heart. A 38-year-old accountant presents to the office with a series of generalized complaints. He relates that he feels a loss of pleasure in daily activities, has difficulty sleeping, and is experiencing problems making decisions. Which of the following best explains the patient's presentation? a. Substance abuse with anhedonia b. Bipolar disorder in the early pre-excitatory phase c. Histrionic personality d. Depression e. Antisocial personality - correct answer d. Depression Concerning hallucinations, an abnormal perception experienced by a patient, which of the following statements is true about this abnormality? a. They include false perceptions associated with dreaming and occurring with falling asleep and awakening. NR509 (3 versions) Midterm exam + NR509 Bates Test Bank - Midterm b. Objective testing can be performed by a trained neuropsychologist to ascertain the correct diagnosis associated with this complaint. c. Although alcoholism may be associated with abnormalities of perception, it is not considered a cause of hallucinations as this finding is due to its direct toxic effects. d. It may occur in association with a number of conditions including delirium and dementia, posttraumatic stress disorder (PTSD), and schizophrenia. e. By definition, hallucinations are confined to those abnormal perceptions that are either auditory or visual in nature. - correct answer d. It may occur in association with a number of conditions including delirium and dementia, posttraumatic stress disorder (PTSD), and schizophrenia. A 24-year-old veteran returns from his second tour of duty in the Middle East. He was witness to a number of violent military encounters and experienced the death of several of his closest friends. He describes a number of problems including nightmares, poor sleep pattern, and mild panic attacks. In persons with trauma- and stress-related disorders as well as other disorders that may be associated with hallucinations and illusions, which of the following statements is true that distinguishes these two entities from each other? a. Illusions occur only when awake, whereas hallucinations can occur both while awake and while sleeping. b. Illusions are a misinterpretation of real stimuli, whereas hallucinations are subjective perceptions in the absence of real stimuli. c. Illusions involve an irrational fear or perceptions, whereas hallucinations are a misinterpretation of real external stimuli. d. Hallucinations may be - correct answer b. Illusions are a misinterpretation of real stimuli, whereas hallucinations are subjective perceptions in the absence of real stimuli. Abstract thinking is an important component of the human thought process. A person's ability to understand questions that test his or her ability to answer appropriately is dependent upon a number of factors. Which one of the following answers is true in identifying a patient with concrete thinking and a reduced ability to think abstractly? NR509 (3 versions) Midterm exam + NR509 Bates Test Bank - Midterm e. Basal cell carcinoma (BCC) - correct answer d. Seborrheic keratosis A 62-year-old manual laborer presents to an annual physical examination with concerns about skin cancer screening. He does not have any lesions of concern but was recently told by a friend that he should have his skin checked by a doctor yearly. What is the best advice for this patient according to the U.S. Preventive Services Task Force (USPSTF) recommendations on skin cancer screening from 2015? a. The USPSTF recommends that all individual age >50 years be screened yearly for skin cancer regardless of risk factors. b. The USPSTF recommendations mirror those of the American Cancer Society (ACS) and American Academy of Dermatologists (AAD) in recommending and annual skin cancer screening for patients age >50 years. c. The USPSTF recommends skin cancer screening only in sun-exposed areas of fair-skinned individuals every 6 months. d. The USPSTF recommends focused screening of individuals with a history of dysplasti - correct answer e. The USPSTF recommends against routine screening for skin cancer due to lack of evidence for this intervention across the general population. A 72-year-old retired woman presents to a primary care provider for evaluation of a suspicious mole. She noticed this lesion 3 weeks ago on her right flank in an area where she had previously seen no abnormality. She is very concerned about melanoma and asks if this could be a possible diagnosis and also wonders if this should have been noticed at her annual examination 7 months ago. Concerning the initial recognition of melanoma, which of the following is true? a. The majority of melanomas are recognized during an annual physical examination. b. Approximately 50% of melanomas are initially noticed by patients then brought to the attention of a practitioner. c. General screening programs conducted by medical facilities identify ~75% of melanomas. d. Most melanomas are initially identified in individuals with positive family histories by DNA analysis for causative genes. e. Asymmetry of a mole is rarely associated wi - correct answer b. Approximately 50% of melanomas are initially noticed by patients then brought to the attention of a practitioner. NR509 (3 versions) Midterm exam + NR509 Bates Test Bank - Midterm A concerned mother brings her 9-year-old daughter to the clinic with several days of a diffuse rash on the trunk. The child was previously healthy and is current on her vaccinations. The mother relates a history of decreased appetite, easy fatigue, and low-grade subjective fevers. On examination, temperature is recorded at 100.5oF, the rash is confirmed as described by the mother, and additional physical findings of a strawberry tongue and erythema of the palms and soles are noted. Nonpainful peeling of the skin of the child's fingertips is noted incidentally. Based on the history and physical findings, which is the most likely diagnosis and course of action? a. Nonspecific viral exanthem, for which observant management is advised b. Contact dermatitis, for which antihistamines are indicated c. Measles, for which review of the vaccination history is critical d. Kawasaki disease, for which close monitoring and possib - correct answer d. Kawasaki disease, for which close monitoring and possibly hospitalization might be required A 16-year-old male high school student presents with a primary concern of acne. He relates a history of 2 years of moderate mild acne and closed comedones (whiteheads), which have recently worsened such that a classmate started calling him a pirate due to a large pustule that developed at the tip of his nose. He has increasing outbreaks of cyst-like acne as well as a generally poor complexion with pitting and scarring from prior outbreaks. Which of the following best describes this condition in the adolescent population? a. Acne vulgaris affects <50% of the adolescent population. b. Acne vulgaris is associated with an identified virus for which there is no definitive treatment. c. Acne vulgaris is associated with blockage of sebaceous glands, stress, humidity, and heavy sweating as well as other contributory factors d. The primary hormonal stimulus for acne vulgaris is estrogen, causing preferentially worse cases in - correct answer c. Acne vulgaris is associated with blockage of sebaceous glands, stress, humidity, and heavy sweating as well as other contributory factors A 72-year-old woman presents with concerns about several ruby-red spots on her chest and abdomen. She reports that these are growing in both size and number over time. On NR509 (3 versions) Midterm exam + NR509 Bates Test Bank - Midterm examination, the provider notes a number of cherry angiomas at the locations indicated by the patient. No other abnormalities are noted. Which of the following best describes the clinical characteristics and significance of a cherry angioma? a. Cherry angiomas never show blanching under pressure. b. Cherry angiomas are associated with liver disease and B vitamin deficiencies. c. Cherry angiomas are benign and may increase in size and number with aging. d. Cherry angiomas rarely occur on the trunk and are most often noted on the legs near veins. e. Cherry angiomas are a marker for underlying pathology that requires additional evaluation. - correct answer c. Cherry angiomas are benign and may increase in size and number with aging. A 28-year-old male business executive presents to a primary care provider with concerns about hair loss. He is otherwise healthy without chronic medical conditions or current medications. He has a chart history of allergy to sulfa medications, although this happened when he was a young child, and he does not recall the incident or the reaction. He is unsure at what age his father went bald, as he never remembers his father having hair. He remarks jokingly that he is losing more hair than his dogs at home, who shed frequently but are otherwise healthy. On examination, he has a single uniform oval patch of hair loss over the left temporal area without any scaling, inflammation, or other skin changes where the hair is missing. Which of the following is the most likely explanation for his hair loss? a. Male pattern baldness, as evidenced by his father's baldness at a young age b. Tinea capitis, as evidenced by his expos - correct answer d. Alopecia areata, as evidenced by patchy hair loss without associated skin changes A 33-year-old nurse presents with a history of weight gain, decreased energy, and menorrhagia over the past several months. Review of her family history reveals Hashimoto thyroiditis and hypothyroidism in four female first-degree relatives (her mother and three sisters). Which of the following skin findings best supports a diagnosis of clinical hypothyroidism? a. Discoid rash, alopecia, oral ulcers, and Raynaud phenomenon b. Warm moist skin, hyperpigmentation, and pretibial myxedema c. Dry skin, myxedema, alopecia of the eyebrows, and brittle nails NR509 (3 versions) Midterm exam + NR509 Bates Test Bank - Midterm pale, saclike growth of inflamed tissue that is obstructing a large part of the nasal cavity. What is the most likely diagnosis? a. Ulcer b. Viral rhinitis c. Allergic rhinitis d. Deviated nasal septum e. Nasal polyp - correct answer e. Nasal polyp 1. A 65-year-old overweight male presents at the clinic with hoarseness which has lasted for around 2 months. He thinks it began along with a cold. He is not feeling badly other than frequent heartburn, and he has continued to work as a bartender (for the past 30 years), but he is having difficulty being heard and understood because of his hoarse voice. A diagnosis that is on the differential list includes which of the following? a. Voice strain from bartending and talking amidst loud ambient noise b. Viral infection c. Acid reflux d. Inhalation of fumes e. Environmental allergies - correct answer c. Acid reflux A 39-year-old architect comes to the clinic for a 2-day history of fever, chills, cough productive of green sputum, and dyspnea. He has no history of serious illness. His temperature is 101.2oF. His other vital signs are within normal limits. Late inspiratory crackles are heard on auscultation over the left lower lung posteriorly. When the clinician listens over that area and instructs the patient to say "ee," it sounds like "A." Which of the following would most likely be found on percussion of his lungs? a. Flatness b. Hyperresonance c. Stridor d. Tympany NR509 (3 versions) Midterm exam + NR509 Bates Test Bank - Midterm e. Dullness - correct answer e. Dullness A student is practicing the performance of a lung examination on a classmate. Which of the following is the correct order for performing the components of the lung examination? a. Auscultation, inspection, palpation, and percussion b. Auscultation, percussion, palpation, and inspection c. Inspection, palpation, percussion, and auscultation d. Auscultation, inspection, palpation, and percussion e. Inspection, auscultation, percussion, and palpation f. Palpation, inspection, auscultation, and percussion - correct answer c. Inspection, palpation, percussion, and auscultation A 14-year-old high school student comes to the clinic for a 3-month history of periodic dyspnea when playing basketball. It resolves shortly after resting. He has not had fever, chills, cough, sputum production, or chest pain. He has no history of serious illness. Based on the boy's history, asthma is suspected. Which of the following sounds heard on expiration during lung auscultation would be most suggestive of asthma? a. Mediastinal crunch b. Pleural rub c. Rhonchi d. Wheezes e. Stridor - correct answer d. Wheezes A clinician is percussing the lungs of a patient with chronic obstructive pulmonary disease to see if they sound hyperresonant. Which of the following is an example of good technique for percussion? a. Strike using the tip of the third finger. b. The proximal interphalangeal joint is the joint that is struck. c. Put the third and fourth fingers next to each other on the chest. NR509 (3 versions) Midterm exam + NR509 Bates Test Bank - Midterm d. Strike using the finger pad of the fourth finger. e. The wrist is kept still during percussion. - correct answer a. Strike using the tip of the third finger. A 29-year-old waiter comes to the clinic for a 2-month history of a cough. When he lowers his gown so the clinician can listen to his lungs, the clinician notices a depression of the lower part of his sternum. Which of the following best describes the appearance of his chest? a. Barrel chest b. Flail chest c. Pectus excavatum d. Pigeon chest e. Thoracic kyphoscoliosis - correct answer c. Pectus excavatum A 13-year-old girl is brought by her mother to the clinic one day before the start of eighth grade because of a 3-day history of episodes of shortness of breath. When she gets the shortness of breath, she also notices tingling around her lips. She has no fever, cough, sputum production, or chest pain. She has no history of serious illness and takes no medications. Vital signs are within normal limits. Cardiac, lung, and extremity examinations show no abnormalities. Which of the following is the most likely diagnosis? a. Aspiration of a foreign body b. Anxiety c. Asthma d. Left-sided heart failure e. Pneumonia - correct answer b. Anxiety A 70-year-old patient has suspected chronic obstructive pulmonary disease. The clinician instructs the patient to take a deep breath in, and then with his mouth open, breathe out as fast and completely as he can. For what is the clinician checking? a. Bronchophony b. Egophony NR509 (3 versions) Midterm exam + NR509 Bates Test Bank - Midterm On routine physical examination, a 40-year-old teacher is found to have a single second heart sound. The most likely explanation for this finding is what? a. Auscultation occurred during inspiration. b. The patient has pulmonic stenosis. c. Auscultation occurred during expiration. d. The patient has a right bundle branch block. e. The patient has a left bundle branch block. - correct answer c. Auscultation occurred during expiration. A first-year medical student is examining a standardized patient with a structurally normal heart. The student is having difficulty auscultating the splitting of the second heart sound. At what area on the patient's chest would the student have the best opportunity of hearing this sound? a. Apex b. Right second interspace c. Left second and third interspace d. Lower left sternal border e. Midsternum - correct answer c. Left second and third interspace A 20-year-old college student is experiencing dyspnea on exertion and palpitations. On cardiac auscultation, the second heart sound is split and fixed on both inspiration and expiration. What is the most likely cardiac condition associated with this finding? a. Left bundle branch block b. Pulmonic stenosis c. Right bundle branch block d. Atrial septal defect e. Tricuspid stenosis - correct answer d. Atrial septal defect NR509 (3 versions) Midterm exam + NR509 Bates Test Bank - Midterm A 70-year-old retired business executive presents to the Emergency Department with progressive shortness of breath and two-pillow orthopnea. On physical examination, the blood pressure is 145/90 mm Hg, there is jugular venous distension, lower extremity pitting edema to the knee, and a blowing holosystolic murmur heard best at the lower left sternal border. No other murmurs or thrills are auscultated on physical exam. Which of the following interventions is to most likely to improve the patient's symptoms? a. Removal of intravascular volume with diuresis b. Replacement of the mitral valve c. Replacement of the aortic valve - correct answer a. Removal of intravascular volume with diuresis A 55-year-old actress sustains a heart attack, and the follow-up electrocardiogram demonstrates a left bundle branch block. What would be the likely duration of the QRS complex? a. 75 milliseconds b. 90 milliseconds c. 95 milliseconds d. 100 milliseconds e. 125 milliseconds - correct answer e. 125 milliseconds A 55-year-old truck driver with obstructive sleep apnea has diastolic heart failure. An echocardiogram demonstrates significant biatrial enlargement. What portion of his electrocardiogram would likely be abnormal? a. QRS complex b. R wave c. S wave d. P wave e. T wave - correct answer d. P wave NR509 (3 versions) Midterm exam + NR509 Bates Test Bank - Midterm A 45-year-old physician is placed on a β-blocker for hypertension. Prior to medication administration, the patient's heart rate is 75 beats per minute with a cardiac output of 5 liters per minute. Following initiation of the medication, the heart rate decreases to 60 beats per minute without a change in stroke volume. What would be the expected new cardiac output? a. 3 liters per minute b. 4 liters per minute c. 5 liters per minute d. 6 liters per minute e. 10 liters per minute - correct answer b. 4 liters per minute A 68-year-old retired administrative assistant complains of a 3-month history of recurring pain after ambulating that radiates from her back in the upper lumbar region into both buttocks, bilateral thighs, and mid-calf regions. Her pain is typically improved by sitting or by leaning forward. The origin of her pain is likely secondary to which of the following? a. Peripheral arterial disease (PAD) b. Venous stasis c. Acute arterial occlusion d. Neurogenic claudication e. Abdominal aortic aneurysm - correct answer d. Neurogenic claudication A patient that has a known history of cardiovascular disease including a myocardial infarction and positive ankle- brachial index indicating peripheral arterial disease in his left leg is now having some issues with erectile dysfunction (ED). The clinician suspects it may be due to medications or further vascular disease. He does not complain of any other symptoms. If his symptoms are related to vascular disease, where would the lesion likely be located? a. Aortorenal b. Iliac pudendal c. Common femoral d. Superficial femoral e. Popliteal - correct answer b. Iliac pudendal NR509 (3 versions) Midterm exam + NR509 Bates Test Bank - Midterm a. Thrombus formation in a superficial vein b. Dilated veins secondary to incompetent valves c. Occluded arterial vessels d. Draining lymphatic channels e. Dilated arterioles - correct answer d. Draining lymphatic channels A clinician, evaluating a patient for valvular competency in the communicating veins of the saphenous system, starts with the patient supine, then elevates one leg to about 90° to empty it of venous blood. Next, the great saphenous vein in the upper part of the thigh is occluded with manual compression, and the patient stands. The clinician keeps the vein occluded while watching for venous filling in the leg. Which test is being performed? a. Allen b. Ankle-brachial index c. Straight-leg raise d. Romberg e. Trendelenburg - correct answer e. Trendelenburg A 44-year-old retail salesperson has noticed an increasing dilatation of the veins in her legs. Upon inspection, it is noted that she has significant varicosities on the posterior aspects of both legs which begin in the lateral side of the foot and pass upward along the posterior calf. The remainder of the veins in the legs appears normal at this time. Which veins are currently affected? a. Great saphenous b. Small saphenous c. Perforating d. Femoral e. Dorsal venous arch - correct answer b. Small saphenous NR509 (3 versions) Midterm exam + NR509 Bates Test Bank - Midterm NR509 Midterm version 3 Sources of Joint Pain - correct answer Nonarticular, intra-articular (Acute <6 weeks), Intra- articular (Chronic, >6 weeks) Non-articular sources - correct answer Trauma/fractures, fibromyalgia, polymyalgia, rheumatica, bursitis, tendinitis Intra-articular (Acute, <6weeks) - correct answer Arthritis (infectious arthritis, gout, pseudo gout, reiter syndrome) Intra-Articular (Chronic, >6weeks) - correct answer Arthritis (inflammatory/non- inflammatory), Lupus, OA, RA, scleroderma, polymyostis What causes saddle numbness? - correct answer Cauda equina syndrome how does retinal detachment present? - correct answer sudden painless unilateral vision loss obtunded means? - correct answer eyes open, slow to respond, slighty confused, decreased interest/alertness in environment cranial nerve in charge of lateral gaze? - correct answer Cranial nerve 6, abducens What should be listed under an adults health history? - correct answer Medical, surgical, OBGYN, psychiatric What conditions do not have a red reflex? - correct answer opacity of the lens (cataracts), vitreous (fake eye), rare but retinal displacement, retinoblastoma in children Signs of seasonal allergies - correct answer Red, runny nose, sneezy, watery, red, and itchy eyes optic neuritis presents by? - correct answer Sudden Painful unilateral vision loss. sometimes associated with MS how does pityriasis rosacea present? - correct answer Oval lesions on trunk, can be in the shape of a christmas tree, especially on children, (Harolds sign)- Largest lesion first What is listed under present illness? - correct answer OLDCART onset, location, duration, characteristics, aggravated factors, relieving factors, and treatment Know where the acromion process is? - correct answer between clavicle and shoulder (Sticks off) NR509 (3 versions) Midterm exam + NR509 Bates Test Bank - Midterm What can cause falsy high BP? - correct answer brachial artery below heart, too small of a cough, white coat syndrome nystagmus (What is it/how to check for it?) - correct answer involuntary movements of the eyes with quick and slow movements. Note the direction of the gaze and the plane of it (horizontal, vertical, rotary, mixed) & the direction of quick and slow movements. it is named for the direction of the quick component. Have patient stare at object across the room and see if nystagmus increases or decreases yellow sclera indicates> - correct answer high billirubin, jaundice signs of degenerative joint disease? - correct answer "giving out sensation" hurts with bending, lifting, twisting, and sitting relieved by changing positions, walking, running, laying down Otosclerosis presents with Weber and Rinne test - correct answer -Weber test: Vibrations are heard in impaired. (room noises not heard well, so detection of vibrations improves) -Rinne Test: BC>=AC. tuning fork at external auditory meatus, then the mastoid bone --- air conduction through the external or middle ear is impaired, vibrations through the bone bypass the problem and reach the cochlea Are cherry angiomas benign or not? - correct answer Benign Visual Acuity score - correct answer 20/40---- 20 is how far the patient is standing from the chart 40--how far normal eyes can read it order for meeting/conducting an interview with patient - correct answer Prepare( reasearch record) Set an agenda check appearances Great and build rapport with patient establish and agenda NR509 (3 versions) Midterm exam + NR509 Bates Test Bank - Midterm respitory distress - correct answer tachypnea, cyanosis, audible sounds, contraction of muscles objective info - correct answer what you detect in the exam, lab values, test data Causes of epistaxis - correct answer trauma (nose picking) crusting/ drying nasal mucousa, tumors, foreign bodies, inflammation otitis externa signs - correct answer tug test. pull on tragus and auricle signs of pneumonia - correct answer dullness replaces resonance crackles plural rubs localized bronchophony and egophony pleuratic pain-sharp knifelike aggrevated by deep inspiration coughing movements fever sputum sings of meningitis - correct answer neck stiffness with resistance to flexion signs of asthma - correct answer cough at times with mucoid sputum, especially near end of attack eposodic wheezing, dyspnea may have history of allergies signs of lyme disease - correct answer circle rash wiht bullseye middle flu-like symptoms ( fever, headache, fatigue) what does acanthosis nigrican clue to - correct answer DM red flags for headaches - correct answer new onset after 50 worse and more frequent in 3 month period sunder onset like thunder calp aggrevated/relieved by position changes precipated by the valsalva maneuver or exerrtion NR509 (3 versions) Midterm exam + NR509 Bates Test Bank - Midterm associated with fever, night sweats, weight loss presence of cancer, HIV, pregnant recent head trauma change in pattern from past headaches assciated with papilledema, neck stiffness, or focal neurological deficits labs to check for vitiligo - correct answer TSH, free T4, free T3 where to sit when intepreter in room - correct answer in front of patient interpreter behind you or beside you what is included in constitutional symptoms - correct answer fatigue, weakness, fever, chills, night sweats, weight change, pain psoriasis presents - correct answer run fingers over .. it raised greater than 1 cm -- plague...less its a papule cotton whool patches look like - correct answer irregular patches seen at diabetic and hypertensive retinopathy subconjunctival hemorrhage - correct answer benign, no treatment necessary, resolves in 2 weeks, leakage of blood outside the vessel producing homogenous, red area, no ocular discharge, vision not affected usually resulting from trauma, sudden increase in venous pressure know to consider angina pectoris as a differentia with CP - correct answer it can be a cause for pain in myocardium. a clenched fist over the sternum suggest angina olfactory cn 1 - correct answer decreased sense of smell if normal in elderly patients, head trauma, smoking, cocaine use, parkinson shoulder shrug test what cranial nerve - correct answer spinal accesorry XI Vasovagal syncope causes - correct answer fear, pain, prolonged sitting, humid environment, fatigue, hunger, dehydration, diuretics, vasodilators NR509 Bates Test Bank - Midterm NR509 (3 versions) Midterm exam + NR509 Bates Test Bank - Midterm A patient presents for evaluation of a sharp, aching chest pain which increases with breathing. Which anatomic area would you localize the symptom to? A) Musculoskeletal B) Reproductive C) Urinary D) Endocrine - correct answer A) Musculoskeletal Chest pain may be due to a musculoskeletal condition, such as costochondritis or intercostal muscle cramp. This would be worsened by motion of the chest wall. Pleuritic chest pain is also a sharp chest pain which increases with a deep breath. This type of pain can occur with inflammation of the pleura from pneumonia or other conditions and pulmonary embolus. A patient comes to the emergency room for evaluation of shortness of breath. To which anatomic region would you assign the symptom? A) Reproductive B) Urinary C) Cardiac D) Hematologic - correct answer C) Cardiac Cardiac disorders such as congestive heart failure are the most likely on this list to result in shortness of breath. There are cases within the other categories which may also result in shortness of breath, such as anemia in the hematologic category, pregnancy in the reproductive category, or sepsis with UTI in the urinary category. A patient presents for evaluation of a cough. Which of the following anatomic regions can be responsible for a cough? NR509 (3 versions) Midterm exam + NR509 Bates Test Bank - Midterm A) Head is normocephalic and atraumatic, fundi with sharp discs, neck supple with full range of motion B) Head is normocephalic and atraumatic, fundi with sharp discs, neck with paraspinous muscle spasm and limited range of motion to the right C) Head is normocephalic and atraumatic, fundi with blurred disc margins, neck tender to palpation, unable to perform range of motion D) Head is normocephalic and atraumatic, fundi with blurred disc margins, neck supple with full range of motion - correct answer C) Head is normocephalic and atraumatic, fundi with blurred disc margins, neck tender to palpation, unable to perform range of motion Blurred disc margins are consistent with papilledema, and neck tenderness and lack of range of motion are consistent with neck stiffness, which in this scenario is likely to be caused by meningeal inflammation. Kernig's and Brudzinski's signs are also helpful in testing for meningeal irritation on exam. A 37-year-old nurse comes for evaluation of colicky right upper quadrant abdominal pain. The pain is associated with nausea and vomiting and occurs 1 to 2 hours after eating greasy foods. Which one of the following physical examination descriptions would be most consistent with the diagnosis of cholecystitis? A) Abdomen is soft, nontender, and nondistended, without hepatosplenomegaly or masses. B) Abdomen is soft and tender to palpation in the right lower quadrant, without rebound or guarding. C) Abdomen is soft and tender to palpation in the right upper quadrant with inspiration, to the point of stopping inspiration, and there is no rebound or guarding. D) Abdomen is soft and tender to palpation in the mid-epigastric area, without rebound or guarding. - correct answer C) Abdomen is soft and tender to palpation in the right upper quadrant with inspiration, to the point of stopping inspiration, and there is no rebound or guarding. NR509 (3 versions) Midterm exam + NR509 Bates Test Bank - Midterm In cholecystitis, the pain, which originates from the gallbladder, is located in the right upper quadrant. Severity of pain with inspiration that is sufficient to stop further inhalation is also known as Murphy's sign, which, if present, is further indicative of inflammation of the gallbladder. A 55-year-old data entry operator comes to the clinic to establish care. She has the following symptoms: headache, neck pain, sinus congestion, sore throat, ringing in ears, sharp brief chest pains at rest, burning abdominal pain with spicy foods, constipation, urinary frequency that is worse with coughing and sneezing, and swelling in legs. This cluster of symptoms is explained by: A) One disease process B) More than one disease process - correct answer B) More than one disease process The patient appears to have several possible conditions: allergic rhinitis, arthritis, conductive hearing loss, pleuritic chest pains, heartburn, stress urinary incontinence, and venous stasis, among other conditions. Although we always try, it is very difficult to assign all of these symptoms to one cohesive diagnosis. A 62-year-old teacher presents to the clinic for evaluation of the following symptoms: fever, headache, sinus congestion, sore throat, green nasal discharge, and cough. This cluster of symptoms is best explained by: A) One disease process B) More than one disease process - correct answer A) One disease process NR509 (3 versions) Midterm exam + NR509 Bates Test Bank - Midterm This cluster of symptoms is most consistent with sinusitis. The chance that all of these symptoms are caused by multiple synchronous conditions in the same patient is much less than the possibility of having one problem which accounts for all of them. Steve has just seen a 5-year-old girl who wheezes when exposed to cats. The patient's family history is positive for asthma. You think the child most likely has asthma. What have you just accomplished? A) You have tested your hypothesis. B) You have developed a plan. C) You have established a working diagnosis. D) You have created a hypothesis. - correct answer D) You have created a hypothesis. As you go through a history and examination, you will start to generate ideas to explain the patient's symptoms. It is best to keep an open mind and make as many hypotheses as you can, to avoid missing a possibility. A common mistake is to latch onto one idea too early. Ms. Washington is a 67-year-old who had a heart attack last month. Now she complains of shortness of breath and not being able to sleep in a flat position (orthopnea). On examination you note increased jugular venous pressure, an S3 gallop, crackles low in the lung fields, and swollen ankles (edema). This is an example of a: A) Pathophysiologic problem B) Psychopathologic problem - correct answer A) Pathophysiologic problem This is an example of a pathophysiologic problem because Ms. Washington's symptoms are consistent with a pathophysiologic process. The heart attack reduced the ability of her heart to handle her volume status and subsequently produced the many features of congestive heart failure. On the way to see your next patient, you glance at the calendar and make a mental note to buy a Mother's Day card. Your patient is Ms. Hernandez, a 76-year-old widow who lost her husband in May, two years ago. She comes in today with a headaches, abdominal pain, and NR509 (3 versions) Midterm exam + NR509 Bates Test Bank - Midterm We generally develop our examinations to fit our clinical experiences. Sensitive tests are performed routinely on the screening examination, while specific tests are usually saved for the detailed or "branched" examinations. Branched examinations are further maneuvers we can perform to investigate positive findings on our screening examinations. Save this type of maneuver to confirm your hypothesis. You have recently returned from a medical missions trip to sub-Saharan Africa, where you learned a great deal about malaria. You decide to use some of the same questions and maneuvers in your "routine" when examining patients in the midwestern United States. You are disappointed to find that despite getting some positive answers and findings, on further workup, none of your patients has malaria except one, who recently emigrated from Ghana. How should you next approach these questions and maneuvers? A) Continue asking these questions in a more selective way. B) Stop asking these questions, because they are low yield. C) Question the validity of the questions. D) Ask these questions of all your patients. - correct answer A) Continue asking these questions in a more selective way. The predictive value of a positive finding depends upon the prevalence of a given disease in a population. The prevalence of malaria in the Midwest is almost zero, except in people immigrating from areas of high prevalence. You will waste time and resources applying these questions and maneuvers to all patients. It would be wise to continue applying what you learned to those who are from areas of high prevalence of a given disease. You will learn to tailor your examination to the population you are serving. For which of the following patients would a comprehensive health history be appropriate? A) A new patient with the chief complaint of "I sprained my ankle". NR509 (3 versions) Midterm exam + NR509 Bates Test Bank - Midterm B) An established patient with the chief complaint of "I have an upper respiratory infection". C) A new patient with the chief complaint of "I am here to establish care". D) A new patient with the chief complaint of "I cut my hand". - correct answer C) A new patient with the chief complaint of "I am here to establish care". This patient is here to establish care, and because she is new to you, a comprehensive health history is appropriate. The components of the health history include all of the following except which one? A) Review of systems B) Thorax and lungs C) Present illness D) Personal and social items - correct answer B) Thorax and lungs The thorax and lungs are part of the physical examination, not part of the health history. Is the following information subjective or objective? Mr. M. has shortness of breath that has persisted for the past 10 days; it is worse with activity and relieved by rest. A) Subjective B) Objective - correct answer A) Subjective This is information given by the patient about the circumstances of his chief complaint. It does not represent an objective observation by the examiner. Is the following information subjective or objective? Mr. M. has a respiratory rate of 32 and a pulse rate of 120. NR509 (3 versions) Midterm exam + NR509 Bates Test Bank - Midterm A) Subjective B) Objective - correct answer B) Objective This is a measurement obtained by the examiner, so it is considered objective data. The patient is unlikely to be able to give this information to the examiner. The following information is recorded in the health history: "The patient has had abdominal pain for 1 week. The pain lasts for 30 minutes at a time; it comes and goes. The severity is 7 to 9 on a scale of 1 to 10. It is accompanied by nausea and vomiting. It is located in the mid-epigastric area." Which of these categories does it belong to? A) Chief complaint B) Present illness C) Personal and social history D) Review of systems - correct answer B) Present illness This information describes the problem of abdominal pain, which is the present illness. The interviewer has obtained the location, timing, severity, and associated manifestations of the pain. The interviewer will still need to obtain information concerning the quality of the pain, the setting in which it occurred, and the factors that aggravate and alleviate the pain. The following information is recorded in the health history: "The patient completed 8th grade. He currently lives with his wife and two children. He works on old cars on the weekend. He works in a glass factory during the week." Which category does it belong to? NR509 (3 versions) Midterm exam + NR509 Bates Test Bank - Midterm never been intubated." A) Adult illnesses B) Surgeries C) Obstetrics/gynecology D) Psychiatric - correct answer A) Adult illnesses This is information about a significant hospitalization and should be placed in the adult illnesses section. If the patient is being seen for an asthma exacerbation, you may consider placing this info in the present illness section, because it relates to the chief complaint at that visit. You are running late after your quarterly quality improvement meeting at the hospital and have just gotten paged from the nurses' station because a family member of one of your patients wants to talk with you about that patient's care. You have clinic this afternoon and are double-booked for the first appointment time; three other patients also have arrived and are sitting in the waiting room. Which of the following demeanors is a behavior consistent with skilled interviewing when you walk into the examination room to speak with your first clinic patient? A) Irritability B) Impatience C) Boredom D) Calm - correct answer D) Calm The appearance of calmness and patience, even when time is limited, is the hallmark of a skilled interviewer. Suzanne, a 25 year old, comes to your clinic to establish care. You are the student preparing NR509 (3 versions) Midterm exam + NR509 Bates Test Bank - Midterm to go into the examination room to interview her. Which of the following is the most logical sequence for the patient-provider interview? A) Establish the agenda, negotiate a plan, establish rapport, and invite the patient's story. B) Invite the patient's story, negotiate a plan, establish the agenda, and establish rapport. C) Greet the patient, establish rapport, invite the patient's story, establish the agenda, expand and clarify the patient's story, and negotiate a plan. D) Negotiate a plan, establish an agenda, invite the patient's story, and establish rapport. - correct answer C) Greet the patient, establish rapport, invite the patient's story, establish the agenda, expand and clarify the patient's story, and negotiate a plan. This is the most productive sequence for the interview. Greeting patients and establishing rapport allows them to feel more comfortable before "inviting" them to relate their story. After hearing the patient's story, together you establish the agenda regarding the most important items to expand upon. At the end, together you negotiate the plan of diagnosis and treatment. Alexandra is a 28-year-old editor who presents to the clinic with abdominal pain. The pain is a dull ache, located in the right upper quadrant, that she rates as a 3 at the least and an 8 at the worst. The pain started a few weeks ago, it lasts for 2 to 3 hours at a time, it comes and goes, and it seems to be worse a couple of hours after eating. She has noticed that it starts after eating greasy foods, so she has cut down on these as much as she can. Initially it occurred once a week, but now it is occurring every other day. Nothing makes it better. From this description, which of the seven attributes of a symptom has been omitted? A) Setting in which the symptom occurs B) Associated manifestations NR509 (3 versions) Midterm exam + NR509 Bates Test Bank - Midterm C) Quality D) Timing - correct answer B) Associated manifestations The interviewer has not recorded whether or not the pain has been accompanied by nausea, vomiting, fever, chills, weight loss, and so on. Associated manifestations are additional symptoms that may accompany the initial chief complaint and that help the examiner to start refining his or her differential diagnosis. Jason is a 41-year-old electrician who presents to the clinic for evaluation of shortness of breath. The shortness of breath occurs with exertion and improves with rest. It has been going on for several months and initially occurred only a couple of times a day with strenuous exertion; however, it has started to occur with minimal exertion and is happening more than a dozen times per day. The shortness of breath lasts for less than 5 minutes at a time. He has no cough, chest pressure, chest pain, swelling in his feet, palpitations, orthopnea, or paroxysmal nocturnal dyspnea. Which of the following symptom attributes was not addressed in this description? A) Severity B) Setting in which the symptom occurs C) Timing D) Associated manifestations - correct answer A) Severity The severity of the symptom was not recorded by the interviewer, so we have no understanding as to how bad the symptom is for this patient. This allows the comparison of pain intensity before and after an intervention. You are interviewing an elderly woman in the ambulatory setting and trying to get more information about her urinary symptoms. Which of the following techniques is not a component NR509 (3 versions) Midterm exam + NR509 Bates Test Bank - Midterm You can also say, "I want to make sure I take good care of this problem because it is very important. We may need to talk about the others at the next appointment. Is that okay with you?" This is a technique that can help you to change the subject but, at the same time, validate the patient's concerns; it also can provide more structure to the interview. Mrs. H. comes to your clinic, wanting antibiotics for a sinus infection. When you enter the room, she appears to be very angry. She has a raised tone of voice and states that she has been waiting for the past hour and has to get back to work. She states that she is unimpressed by the reception staff, the nurse, and the clinic in general and wants to know why the office wouldn't call in an antibiotic for her. Which of the following techniques is not useful in helping to calm this patient? A) Avoiding admission that you had a part in provoking her anger because you were late B) Accepting angry feelings from the patient and trying not to get angry in return C) Staying calm D) Keeping your posture relaxed - correct answer A) Avoiding admission that you had a part in provoking her anger because you were late In this scenario, the provider was 1 hour late in seeing the patient. The provider should acknowledge that he was late and apologize for this, no matter the reason for being late. It often helps to acknowledge that a patient's anger with you is understandable and that you might be angry in a similar situation. A 23-year-old graduate student comes to your clinic for evaluation of a urethral discharge. As the provider, you need to get a sexual history. Which one of the following questions is inappropriate for eliciting the information? A) Are you sexually active? B) When was the last time you had intimate physical contact with someone, and did that contact include sexual intercourse? C) Do you have sex with men, women, or both? NR509 (3 versions) Midterm exam + NR509 Bates Test Bank - Midterm D) How many sexual partners have you had in the last 6 months? - correct answer A) Are you sexually active? This is inappropriate because it is too vague. Given the complaint, you should probably assume that he is sexually active. A specific sexual history will help you to assess this patient's risk for other sexually transmitted infections. Mr. Q. is a 45-year-old salesman who comes to your office for evaluation of fatigue. He has come to the office many times in the past with a variety of injuries, and you suspect that he has a problem with alcohol. Which one of the following questions will be most helpful in diagnosing this problem? A) You are an alcoholic, aren't you? B) When was your last drink? C) Do you drink 2 to 3 beers every weekend? D) Do you drink alcohol when you are supposed to be working? - correct answer B) When was your last drink? This is a good opening question that is general and neutral in tone; depending on the timing, you will be able to ask for more specific information related to the patient's last drink. The others will tend to stifle the conversation because they are closed-ended questions. On a very busy day in the office, Mrs. Donelan, who is 81 years old, comes for her usual visit for her blood pressure. She is on a low-dose diuretic chronically and denies any side effects. Her blood pressure is 118/78 today, which is well-controlled. As you are writing her script, she mentions that it is hard not having her husband Bill around anymore. What would you do next? A) Hand her the script and make sure she has a 3-month follow-up appointment. B) Make sure she understands the script. C) Ask why Bill is not there. NR509 (3 versions) Midterm exam + NR509 Bates Test Bank - Midterm D) Explain that you will have more time at the next visit to discuss this. - correct answer C) Ask why Bill is not there. Sometimes, the patient's greatest need is for support and empathy. It would be inappropriate to ignore this comment today. She may have relied heavily upon Bill for care and may be in danger. She may be depressed and even suicidal, but you will not know unless you discuss this with her. Most importantly, you should empathize with her by saying something like "It must be very difficult not to have him at home" and allow a pause for her to answer. You may also ask "What did you rely on him to do for you?" A patient is describing a very personal part of her history very quickly and in great detail. How should you react to this? A) Write down as much as you can, as quickly as possible. B) Ask her to repeat key phrases or to pause at regular intervals, so you can get almost every word. C) Tell her that she can go over the notes later to make sure they are accurate. D) Push away from the keyboard or put down your pen and listen. - correct answer D) Push away from the keyboard or put down your pen and listen. This is a common event in clinical practice. It is much more important to listen actively with good eye contact at this time than to document the story verbatim. You want to minimize interruption (e.g., answer B). It is usually not appropriate to ask a patient to go over the written notes, but it would be a good idea to repeat the main ideas back to her. You should be certain she has completed her story before doing this. You arrive at the bedside of an elderly woman who has had a stroke, affecting her entire right side. She cannot speak (aphasia). You are supposed to examine her. You notice that NR509 (3 versions) Midterm exam + NR509 Bates Test Bank - Midterm You comment, "That must have been a very trying time for you." What is this an example of? A) Reassurance B) Empathy C) Summarization D) Validation - correct answer D) Validation This is an example of validation to legitimize her emotional experience. You are performing a young woman's first pelvic examination. You make sure to tell her verbally what is coming next and what to expect. Then you carry out each maneuver of the examination. You let her know at the outset that if she needs a break or wants to stop, this is possible. You ask several times during the examination, "How are you doing, Brittney?" What are you accomplishing with these techniques? A) Increasing the patient's sense of control B) Increasing the patient's trust in you as a caregiver C) Decreasing her sense of vulnerability D) All of the above - correct answer D) All of the above These techniques minimize the effects of transitions during an examination and empower the patient. Especially during a sensitive examination, it is important to give the patient as much control as possible. When using an interpreter to facilitate an interview, where should the interpreter be positioned? A) Behind you, the examiner, so that the lips of the patient and the patient's nonverbal cues can be seen NR509 (3 versions) Midterm exam + NR509 Bates Test Bank - Midterm B) Next to the patient, so the examiner can maintain eye contact and observe the nonverbal cues of the patient C) Between you and the patient so all parties can make the necessary observations D) In a corner of the room so as to provide minimal distraction to the interview - correct answer B) Next to the patient, so the examiner can maintain eye contact and observe the nonverbal cues of the patient Interpreters are invaluable in encounters where the examiner and patient do not speak the same language, including encounters with the deaf. It should be noted that deaf people from different regions of the world use different sign languages. The priority is for you to have a good view of the patient. A 15-year-old high school sophomore and her mother come to your clinic because the mother is concerned about her daughter's weight. You measure her daughter's height and weight and obtain a BMI of 19.5 kg/m2. Based on this information, which of the following is appropriate? A) Refer the patient to a nutritionist and a psychologist because the patient is anorexic. B) Reassure the mother that this is a normal body weight. C) Give the patient information about exercise because the patient is obese. D) Give the patient information concerning reduction of fat and cholesterol in her diet because she is obese. - correct answer B) Reassure the mother that this is a normal body weight. The patient has a normal BMI; the range for a normal BMI is 18.5 to 24.9 kg/m2. You may be able to give the patient and her mother the lower limit of normal in pounds for her daughter's height, or instruct her in how to use a BMI table. A 25-year-old radio announcer comes to the clinic for an annual examination. His BMI is NR509 (3 versions) Midterm exam + NR509 Bates Test Bank - Midterm 26.0 kg/m2. He is concerned about his weight. Based on this information, what is appropriate counsel for the patient during the visit? A) Refer the patient to a nutritionist because he is anorexic. B) Reassure the patient that he has a normal body weight. C) Give the patient information about reduction of fat, cholesterol, and calories because he is overweight. D) Give the patient information about reduction of fat and cholesterol because he is obese. - correct answer C) Give the patient information about reduction of fat, cholesterol, and calories because he is overweight. The patient has a BMI in the overweight range, which is 25.0 to 29.9 kg/m2. It is prudent to give him information about reducing calories, fat, and cholesterol in his diet to help prevent further weight gain. A 30-year-old sales clerk comes to your office wanting to lose weight; her BMI is 30.0 kg/m2. What is the most appropriate amount for a weekly weight reduction goal? A) .5 to 1 pound per week B) 1 to 2.5 pounds per week C) 2.5 to 3.5 pounds per week D) 3.5 to 4.5 pounds per week - correct answer A) .5 to 1 pound per week Based on the NIH Obesity Guidelines, this is the weekly weight loss goal to strive for to maintain long-term control of weight. More rapid weight loss than this does not result in a better outcome at one year. A 67-year-old retired janitor comes to the clinic with his wife. She brought him in because she is concerned about his weight loss. He has a history of smoking 3 packs of cigarettes a day for 30 years, for a total of 90 pack-years. He has noticed a daily cough for the past several NR509 (3 versions) Midterm exam + NR509 Bates Test Bank - Midterm D) Bigeminal - correct answer B) Small amplitude, weak Congestive heart failure is characterized by decreased stroke volume or increased peripheral vascular resistance, which would result in a small-amplitude, weak pulse. Subtle differences in amplitude are usually best detected in large arteries close to the heart, like the carotid pulse. You may not be able to notice these in other locations. An 18-year-old college freshman presents to the clinic for evaluation of gastroenteritis. You measure the patient's temperature and it is 104 degrees Fahrenheit. What type of pulse would you expect to feel during his initial examination? A) Large amplitude, forceful B) Small amplitude, weak C) Normal D) Bigeminal - correct answer A) Large amplitude, forceful Fever results in an increased stroke volume, which results in a large-amplitude, forceful pulse. Later in the course of the illness, if dehydration and shock result, you may expect small amplitude and weak pulses. A 25-year-old type 1 diabetic clerk presents to the emergency room with shortness of breath and states that his blood sugar was 605 at home. You diagnose the patient with diabetic ketoacidosis. What is the expected pattern of breathing? A) Normal B) Rapid and shallow C) Rapid and deep D) Slow - correct answer C) Rapid and deep NR509 (3 versions) Midterm exam + NR509 Bates Test Bank - Midterm The body is trying to rid itself of carbon dioxide to compensate for the acidosis. This is known as Kussmaul's breathing and is seen in other causes of acidosis as well. Mrs. Lenzo weighs herself every day with a very accurate balance-type scale. She has noticed that over the past 2 days she has gained 4 pounds. How would you best explain this? A) Attribute this to some overeating at the holidays. B) Attribute this to wearing different clothing. C) Attribute this to body fluid. D) Attribute this to instrument inaccuracy. - correct answer C) Attribute this to body fluid. This amount of weight over a short period should make one think of body fluid changes. You may consider a kidney problem or heart failure in your differential. The other reasons should be considered as well, but this amount of weight gain over a short period usually indicates causes other than excessive caloric intake. Mr. Curtiss has a history of obesity, diabetes, osteoarthritis of the knees, HTN, and obstructive sleep apnea. His BMI is 43 and he has been discouraged by his difficulty in losing weight. He is also discouraged that his goal weight is 158 pounds away. What would you tell him? A) "When you get down to your goal weight, you will feel so much better." B) "Some people seem to be able to lose weight and others just can't, no matter how hard they try." C) "We are coming up with new medicines and methods to treat your conditions every day." D) "Even a weight loss of 10% can make a noticeable improvement in the problems you mention." - correct answer D) "Even a weight loss of 10% can make a noticeable improvement in the problems you NR509 (3 versions) Midterm exam + NR509 Bates Test Bank - Midterm mention." Many patients trying to change a habit are overwhelmed by how far they are from their goal. Many patients find it empowering to know that they can achieve a small goal, such as a loss of 1 pound per week. Research has shown that significant benefits often come with even a 10% weight loss. Jenny is one of your favorite patients who usually shares a joke with you and is nattily dressed. Today she is dressed in old jeans, lacks makeup, and avoids eye contact. To what do you attribute these changes? A) She is lacking sleep. B) She is fatigued from work. C) She is running into financial difficulty. D) She is depressed. - correct answer D) She is depressed. It is important to use all of your skills and memory of an individual patient to guide your thought process. She is not described as sleepy. Work fatigue would most likely not cause avoidance of eye contact. Financial difficulties would not necessarily deplete a nice wardrobe. It is most likely that she is depressed or in another type of difficulty. You are seeing an older patient who has not had medical care for many years. Her vital signs taken by your office staff are: T 37.2, HR 78, BP 118/92, and RR 14, and she denies pain. You notice that she has some hypertensive changes in her retinas and you find mild proteinuria on a urine test in your office. You expected the BP to be higher. She is not on any medications. What do you think is causing this BP reading, which doesn't correlate with the other findings? A) It is caused by an "auscultatory gap." B) It is caused by a cuff size error. NR509 (3 versions) Midterm exam + NR509 Bates Test Bank - Midterm because of obesity. What should you do next? A) Explain that even small amounts of weight gain can classify you as obese. B) Place him on a high-protein, low-fat diet. C) Advise him to increase his aerobic exercise for calorie burning. D) Measure his waist. - correct answer D) Measure his waist. The patient most likely had a high BMI because of increased muscle mass. In this situation, it is important to measure his waist. It is most likely under 40 inches, which makes obesity unlikely (even to an insurance company). It is important that you personally contact the company and explain your reasoning. Ms. Wright comes to your office, complaining of palpitations. While checking her pulse you notice an irregular rhythm. When you listen to her heart, every fourth beat sounds different. It sounds like a triplet rather than the usual "lub dup." How would you document your examination? A) Regular rate and rhythm B) Irregularly irregular rhythm C) Regularly irregular rhythm D) Bradycardia - correct answer C) Regularly irregular rhythm Because this unusual beat occurs every fourth set of heart sounds, it is regularly irregular. This is most consistent with premature ventricular contractions (or PVCs). This is generally a common and benign rhythm. A 19-year-old college student, Todd, is brought to your clinic by his mother. She is concerned that there is something seriously wrong with him. She states for the past 6 months his behavior has become peculiar and he has flunked out of college. Todd denies any recent illness or injuries. His past medical history is remarkable only for a broken foot. His parents are both healthy. He has a paternal uncle who had similar symptoms in college. The patient admits to NR509 (3 versions) Midterm exam + NR509 Bates Test Bank - Midterm smoking cigarettes and drinking alcohol. He also admits to marijuana use but none in the last week. He denies using any other substances. He denies any feelings of depression or anxiety. While speaking with Todd and his mother you do a complete physical examination, which is essentially normal. When you question him on how he is feeling, he says that he is very worried that Microsoft has stolen his software for creating a better browser. He tells you he has se - correct answer D) Schizophrenia Schizophrenia generally occurs in the late teens to early 20s. It often is seen in other family members, as in this case. Symptoms must be present for at least 6 months and must have at least two features of (1) delusions (e.g., Microsoft is after his programs), (2) hallucinations (e.g., technicians sending telepathic signals), (3) disorganized speech, (4) disorganized behavior, and (5) negative symptoms such as a flat affect. A 24-year-old secretary comes to your clinic, complaining of difficulty sleeping, severe nightmares, and irritability. She states it all began 6 months ago when she went to a fast food restaurant at midnight. While she was waiting in her car a man entered through the passenger door and put a gun to her head. He had her drive to a remote area, where he took her money and threatened to kill her. When the gun jammed he panicked and ran off. Ever since this occurred the patient has been having these symptoms. She states she jumps at every noise and refuses to drive at night. She states her anxiety has had such a marked influence on her job performance she is afraid she will be fired. She denies any recent illnesses or injuries. Her past medical history is unremarkable. On examination you find a nervous woman appearing her stated age. Her physical examination is unremarkable. You recommend medication and counseling. What - correct answer C) Post-traumatic stress disorder Post-traumatic stress disorder is the fearful response (nightmares, avoidance of NR509 (3 versions) Midterm exam + NR509 Bates Test Bank - Midterm areas, irritability) to an event that occurred at least 1 month prior to presentation. The patient's fears and reactions cause marked distress and impair social and occupational functions. A 75-year-old homemaker brings her 76-year-old husband to your clinic. She states that 4 months ago he had a stroke and ever since she has been frustrated with his problems with communication. They were at a restaurant after church one Sunday when he suddenly became quiet. When she realized something was wrong he was taken to the hospital by EMS. He spent 2 weeks in the hospital with right-sided weakness and difficulty speaking. After hospitalization he was in a rehab center, where he regained the ability to walk and most of the use of his right hand. He also began to speak more, but she says that much of the time "he doesn't make any sense." She gives an example that when she reminded him the car needed to be serviced he told her "I will change the Kool-Aid out of the sink myself with the ludrip." She says that these sayings are becoming frustrating. She wants you to tell her what is wrong and what you can do about - correct answer A) Wernicke's aphasia With Wernicke's aphasia the patient can speak effortlessly and fluently, but his words often make no sense. Words can be malformed or completely invented. Wernicke's area is found on the temporal lobes. Susanne is a 27 year old who has had headaches, muscle aches, and fatigue for the last 2 months. You have completed a thorough history, examination, and laboratory workup but have not found a cause. What would your next action be? A) A referral to a neurologist B) A referral to a rheumatologist C) To tell the patient you can't find anything D) To screen for depression - correct answer D) To screen for depression NR509 (3 versions) Midterm exam + NR509 Bates Test Bank - Midterm are a variety of erythematous papules and macules on the cleavage lines of the back. Based on this description, what is the most likely diagnosis? A) Pityriasis rosea B) Tinea versicolor C) Psoriasis D) Atopic eczema - correct answer A) Pityriasis rosea This is a classic description of pityriasis rosea. The description of a large single or "herald" patch preceding the eruption is a good way to distinguish this rash from other conditions. A 19-year-old construction worker presents for evaluation of a rash. He notes that it started on his back with a multitude of spots and is also on his arms, chest, and neck. It itches a lot. He does sweat more than before because being outdoors is part of his job. On physical examination, you note dark tan patches with a reddish cast that has sharp borders and fine scales, scattered more prominently around the upper back, chest, neck, and upper arms as well as under the arms. Based on this description, what is your most likely diagnosis? A) Pityriasis rosea B) Tinea versicolor C) Psoriasis D) Atopic eczema - correct answer B) Tinea versicolor This is a typical description of tinea versicolor. The information that the patient is sweating more also helps support this diagnosis, because tinea is a fungal infection and is promoted by moisture. A 68-year-old retired farmer comes to your office for evaluation of a skin lesion. On the NR509 (3 versions) Midterm exam + NR509 Bates Test Bank - Midterm right temporal area of the forehead, you see a flattened papule the same color as his skin, covered by a dry scale that is round and feels hard. He has several more of these scattered on the forehead, arms, and legs. Based on this description, what is your most likely diagnosis? A) Actinic keratosis B) Seborrheic keratosis C) Basal cell carcinoma D) Squamous cell carcinoma - correct answer This is a typical description of actinic keratosis. Actinic keratosis may be easier to feel than to see. If left untreated, approximately 1% of cases can develop into squamous cell carcinoma. A 58-year-old gardener comes to your office for evaluation of a new lesion on her upper chest. The lesion appears to be "stuck on" and is oval, brown, and slightly elevated with a flat surface. It has a rough, wartlike texture on palpation. Based on this description, what is your most likely diagnosis? A) Actinic keratosis B) Seborrheic keratosis C) Basal cell carcinoma D) Squamous cell carcinoma - correct answer B) Seborrheic keratosis This is a typical description for seborrheic keratosis. The "stuck on" appearance and the rough, wartlike texture are key features for the diagnosis. They often produce a greasy scale when scratched with a fingernail, which further helps to distinguish them from other lesions. Frequently, these benign lesions actually meet several of the ABCDEs of melanoma, so it is important to distinguish these lesions to prevent unnecessary biopsy. A 72-year-old teacher comes to a skilled nursing facility for rehabilitation after being in the hospital for 6 weeks. She was treated for sepsis and respiratory failure and had to be on the ventilator for 3 weeks. You are completing your initial assessment and are evaluating her skin NR509 (3 versions) Midterm exam + NR509 Bates Test Bank - Midterm condition. On her sacrum there is full-thickness skin loss that is 5 cm in diameter, with damage to the subcutaneous tissue. The underlying muscle is not affected. You diagnose this as a pressure ulcer. What is the stage of this ulcer? A) Stage 1 B) Stage 2 C) Stage 3 D) Stage 4 - correct answer C) Stage 3 A stage 3 ulcer is a full-thickness skin loss with damage to or necrosis of subcutaneous tissue that may extend to, but not through, the underlying muscle. An 8-year-old girl comes with her mother for evaluation of hair loss. She denies pulling or twisting her hair, and her mother has not noted this behavior at all. She does not put her hair in braids. On physical examination, you note a clearly demarcated, round patch of hair loss without visible scaling or inflammation. There are no hair shafts visible. Based on this description, what is your most likely diagnosis? A) Alopecia areata B) Trichotillomania C) Tinea capitis D) Traction alopecia - correct answer A) Alopecia areata This is a typical description for alopecia areata. There are no risk factors for trichotillomania or for traction alopecia. The physical examination is not consistent with tinea capitis because the skin is intact. A mother brings her 11 month old to you because her mother-in-law and others have told her that her baby is jaundiced. She is eating and growing well and performing the developmental milestones she should for her age. On examination you indeed notice a yellow tone to her skin from head to toe. Her sclerae are white. To which area should your next questions be related? NR509 (3 versions) Midterm exam + NR509 Bates Test Bank - Midterm Mrs. Hill is a 28-year-old African-American with a history of SLE (systemic lupus erythematosus). She has noticed a raised, dark red rash on her legs. When you press on the rash, it doesn't blanch. What would you tell her regarding her rash? A) It is likely to be related to her lupus. B) It is likely to be related to an exposure to a chemical. C) It is likely to be related to an allergic reaction. D) It should not cause any problems. - correct answer A) It is likely to be related to her lupus. A "palpable purpura" is usually associated with a vasculitis. This is an inflammatory condition of the blood vessels often associated with systemic rheumatic disease. It can cut off circulation to any portion of the body and can mimic many other diseases in this manner. While allergic and chemical exposures may be a possible cause of the rash, this patient's SLE should make you consider vasculitis. Jacob, a 33-year-old construction worker, complains of a "lump on his back" over his scapula. It has been there for about a year and is getting larger. He says his wife has been able to squeeze out a cheesy-textured substance on occasion. He worries this may be cancer. When gently pinched from the side, a prominent dimple forms in the middle of the mass. What is most likely? A) An enlarged lymph node B) A sebaceous cyst C) An actinic keratosis D) A malignant lesion - correct answer B) A sebaceous cyst This is a classic description of an epidermal inclusion cyst resulting from a blocked sebaceous gland. The fact that any lesion is enlarging is worrisome, but the other descriptors are so distinctive that cancer is highly unlikely. This would be an unusual location for a lymph node, and these do not usually drain to the skin. NR509 (3 versions) Midterm exam + NR509 Bates Test Bank - Midterm A young man comes to you with an extremely pruritic rash over his knees and elbows which has come and gone for several years. It seems to be worse in the winter and improves with some sun exposure. On examination, you notice scabbing and crusting with some silvery scale, and you are observant enough to notice small "pits" in his nails. What would account for these findings? A) Eczema B) Pityriasis rosea C) Psoriasis D) Tinea infection - correct answer C) Psoriasis This is a classic presentation of plaque psoriasis. Eczema is usually over the flexor surfaces and does not scale, whereas psoriasis affects the extensor surfaces. Pityriasis usually is limited to the trunk and proximal extremities. Tinea has a much finer scale associated with it, almost like powder, and is found in dark and moist areas. Mrs. Anderson presents with an itchy rash which is raised and appears and disappears in various locations. Each lesion lasts for many minutes. What most likely accounts for this rash? A) Insect bites B) Urticaria, or hives C) Psoriasis D) Purpura - correct answer B) Urticaria, or hives This is a typical case of urticaria. The most unusual aspect of this condition is that the lesions "move" from place to place. This would be distinctly unusual for the other causes listed. NR509 (3 versions) Midterm exam + NR509 Bates Test Bank - Midterm Ms. Whiting is a 68 year old who comes in for her usual follow-up visit. You notice a few flat red and purple lesions, about 6 cm in diameter, on the ulnar aspect of her forearms but nowhere else. She doesn't mention them. They are tender when you examine them. What should you do? A) Conclude that these are lesions she has had for a long time. B) Wait for her to mention them before asking further questions. C) Ask how she acquired them. D) Conduct the visit as usual for the patient. - correct answer C) Ask how she acquired them. These are consistent with ecchymoses, or bruises. It is important to ask about antiplatelet medications such as aspirin, trauma history, and history of blood disorders in the patient and her family. Because of the different ages of the bruises and the isolation of them to the ulnar forearms, these may be a result of abuse or other violence. It is your duty to investigate the cause of these lesions. A middle-aged man comes in because he has noticed multiple small, blood-red, raised lesions over his anterior chest and abdomen for the past several months. They are not painful and he has not noted any bleeding or bruising. He is concerned this may be consistent with a dangerous condition. What should you do? A) Reassure him that there is nothing to worry about. B) Do laboratory work to check for platelet problems. C) Obtain an extensive history regarding blood problems and bleeding disorders. D) Do a skin biopsy in the office. - correct answer A) Reassure him that there is nothing to worry about. These represent cherry angiomas, which are very common, benign lesions. Further workup such as laboratory work, skin biopsy, or even further questions are not necessary at this time. It would be wise to ask the patient to report any changes in any of his skin lesions, and